¿Cómo podemos vincular la probabilidad de que una variable aleatoria sea máxima?


21

Supongamos que tenemos variables aleatorias independientes X_1 , \ ldots , X_n con medios finitos \ mu_1 \ leq \ ldots \ leq \ mu_N y varianzas \ sigma_1 ^ 2 , \ ldots , \ sigma_N ^ 2 . Estoy buscando límites sin distribución en la probabilidad de que cualquier X_i \ neq X_N sea ​​mayor que el resto de X_j , j \ neq i .norteX1...Xnorteμ1...μnorteσ12...σnorte2XyoXnorteXjjyo

En otras palabras, si por simplicidad asumimos que las distribuciones de Xyo son continuas (tal que PAGS(Xyo=Xj)=0 0 ), estoy buscando límites en:

PAGS(Xyo=maxjXj).
Si norte=2 , podemos usar la desigualdad de Chebyshev para obtener:
PAGS(X1=maxjXj)=PAGS(X1>X2)σ12+σ22σ12+σ22+(μ1-μ2)2.
Me gustaría encontrar algunos límites simples (no necesariamente ajustados) para el N general norte, pero no he podido encontrar resultados (estéticamente) agradables para el N general norte.

Tenga en cuenta que no se supone que las variables sean iid. Cualquier sugerencia o referencia al trabajo relacionado es bienvenida.


Actualización: recuerde que por suposición, μjμyo . Entonces podemos usar el límite anterior para llegar a:

PAGS(Xyo=maxjXj)minj>yoσyo2+σj2σyo2+σj2+(μj-μyo)2σyo2+σnorte2σyo2+σnorte2+(μnorte-μyo)2.
Esto implica:
(μnorte-μyo)PAGS(Xyo=maxjXj)(μnorte-μyo)σyo2+σnorte2σyo2+σnorte2+(μnorte-μyo)212σyo2+σnorte2.
Esto, a su vez, implica:
yo=1norteμyoPAGS(Xyo=maxjXj)μnorte-norte2yo=1norte-1(σyo2+σnorte2).
Ahora estoy preguntando si esta ligado puede ser mejorada a algo que no depende linealmente de norte . Por ejemplo, se cumple lo siguiente:
yo=1norteμyoPAGS(Xyo=maxjXj)μnorte-yo=1norteσyo2?
Y si no, ¿qué podría ser un contraejemplo?

3
Esta cota pueden ser más estrictas si se utiliza el índice que proporciona la menor cota superior en lugar de . Tenga en cuenta que este valor depende tanto de la media como de la varianza. jnorte

55
@MichaelChernick: No creo que eso sea correcto. Supongamos, por ejemplo, que tenemos tres distribuciones uniformes en . Entonces, si no me equivoco, , mientras que . No sé si escribir , pero el mismo ejemplo muestra que todavía no es válido. P ( X 1 < max j X j ) = 2 / 3 P ( X 1 < X 2 ) = P ( X 1 < X 3 ) = 1 / 2 P ( X i > max j X j )[0 0,1]PAGS(X1<maxjXj)=2/ /3PAGS(X1<X2)=PAGS(X1<X3)=1/ /2PAGS(Xyo>maxjXj)
MLS

2
@ Michael: Eso todavía no es cierto, desafortunadamente. Los eventos para el fijo no son independientes. iUNAj={Xyo>Xj} yo
cardenal

2
@cardinal: Entre otras cosas, está relacionado con bandidos con múltiples brazos. Si elige un brazo en función de las recompensas anteriores, ¿qué tan grande es la probabilidad de que haya elegido el mejor brazo (que sería en la notación anterior), y podemos limitar la pérdida esperada por elegir un sub -brazo óptimo? P(XN=maxjXj)
MLS

2
Crossposted to MathOverflow: mathoverflow.net/questions/99313
cardenal

Respuestas:


1

Puede usar la desigualdad multivariada de Chebyshev.

Caso de dos variables

Para una situación única, vs , llego a la misma situación que el comentario de Jochen el 4 de noviembre de 2016X1X2

1) Si entonces P ( X 1 > X 2 ) ( σ 2 1 + σ 2 2 ) / ( μ 1 - μ 2 ) 2μ1<μ2P(X1>X2)(σ12+σ22)/(μ1μ2)2

(y también me pregunto acerca de tu derivación)

Derivación de la ecuación 1

  • usando la nueva variableX1X2
  • transformándolo de modo que tenga la media en cero
  • tomando el valor absoluto
  • aplicando la desigualdad de Chebyshev

PAGS(X1>X2)=PAGS(X1-X2>0 0)=PAGS(X1-X2-(μ1-μ2)>-(μ1-μ2))PAGS(El |X1-X2-(μ1-μ2)El |>μ2-μ1)σ(X1-X2-(μ1-μ2))2(μ2-μ1)2=σX12+σX22(μ2-μ1)2

Caso multivariante

La desigualdad en la ecuación (1) se puede cambiar a un caso multivariado al aplicarla a múltiples variables transformadas para cada (tenga en cuenta que están correlacionadas).i < n(XnXi)i<n

I. Olkin y JW Pratt han descrito una solución a este problema (multivariante y correlacionado). 'Una desigualdad de Tchebycheff multivariante' en Annals of Mathematical Statistics, volumen 29 páginas 226-234 http://projecteuclid.org/euclid.aoms/1177706720

Note el teorema 2.3

P(|yi|kiσi for some i)=P(|xi|1 for some i)(u+(ptu)(p1))2p2

en el que el número de variables, , y .t = k - 2 i u = ρ i j / ( k i k j )pt=ki2u=ρij/(kikj)

El teorema 3.6 proporciona un límite más estricto, pero es menos fácil de calcular.

Editar

Se puede encontrar un límite más agudo utilizando la desigualdad multivariada de Cantelli . Esa desigualdad es el tipo que usó antes y le proporcionó el límite que es más nítido que .( σ 2 1 + σ 2 2 ) / ( μ 1 - μ 2 ) 2(σ12+σ22)/(σ12+σ22+(μ1μ2)2)(σ12+σ22)/(μ1μ2)2

No me he tomado el tiempo de estudiar todo el artículo, pero de todos modos, puedes encontrar una solución aquí:

AW Marshall e I. Olkin 'Una desigualdad unilateral del tipo Chebyshev' en Annals of Mathematical Statistics volumen 31 págs. 488-491 https://projecteuclid.org/euclid.aoms/1177705913

(nota posterior: esta desigualdad es para correlaciones iguales y no ayuda suficiente. Pero de todos modos su problema, para encontrar el límite más agudo, es igual a la desigualdad de Cantelli, más general y multivariante. Me sorprendería si la solución no existe)


¿Podría proporcionar una declaración clara de la desigualdad multivariada de Chebyshev?
whuber

1
He editado la solución proporcionando el teorema completo.
Sextus Empiricus

-1

He encontrado un teorema que podría ayudarte y trataré de ajustarlo a tus necesidades. Suponga que tiene:

exp(tE(max1inXi))

Luego, por la desigualdad de Jensen (ya que exp (.) Es una función convexa), obtenemos:

exp(tE(max1inXi))E(exp(tmax1inXi))=E(max1in exp(tXi))i=1nE(exp(tXi)

Ahora para tienes que enchufar en cualquier momento que sea la función generadora de tu variable aleatoria (ya que es solo la definición de mgf). Luego, después de hacerlo (y potencialmente simplificando su término), tome este término y tome el registro y divídalo entre t para obtener una declaración sobre el término . Entonces puede elegir t con algún valor arbitrario (mejor para que el término sea pequeño y el límite sea ajustado).exp(tXiXiE(max1inXi)

Luego, tiene una declaración sobre el valor esperado del máximo sobre n rvs. Para obtener ahora una declaración sobre la probabilidad de que el máximo de esos rv se desvíe de este valor esperado, puede usar la desigualdad de Markov (suponiendo que su rv no es negativo) u otro rv más específico, que se aplica a su rv particular.

Al usar nuestro sitio, usted reconoce que ha leído y comprende nuestra Política de Cookies y Política de Privacidad.
Licensed under cc by-sa 3.0 with attribution required.